Minimiser fonction à plusieurs variables

Bonjour je viens pour avoir un peu d'aide pour un exercice assez simple.

J'ai un ensemble de réels x1, x2,...,xn avec n un entier.
Je connais la somme de x1+x2+...+xn qui est égale à 1.
et je connais les valeurs de a1, a2,...,an qui sont réels aussi et tous positifs, je dois trouver les valeurs de x1,,..,n telle que :
x12a1 + ... + xn2an
soit le plus petit possible (la fonction à minimiser), j'arrive assez facilement lorsque n=2 (en remplaçant x1 par 1 - x2 puis en annulant la dérivé par apport à x2) mais je n'y arrive pas pour tout n.

(il faut que j'arrive à exprimer les valeurs de x1,..,n en fonction de celle de a1,..,n).
Merci de vôtre aide si quelqu'un a une piste :-)

Réponses

  • Bonjour,

    Tu n’as pas écrit ce que sont les $x$ et les $a$. L’écriture avec $*$ est incompréhensible. Quelle est la fonction à minimiser ?
  • Merci, j'ai modifié mon post initial pour rendre le problème plus compréhensible j'espère que c'est mieux.
  • Bonjour,

    Classe les $x$ par ordre croissant.

    Classe les $a$ par ordre croissant.

    Montre que $\displaystyle \sum x^2 a\geq {1\over n} \sum x^2 \sum a.$

    Je ne suis pas sûr que tu as bien défini le problème. Si les $x$ et les $a$ sont donnés, mon indication devrait s’approcher du minimum.
  • merci de ton aide, les x ne sont pas donner il s'agit justement de trouver les x en fonction des a. (en formule général car ils ne sont pas données de façon explicite).
  • Bonjour Louis293.

    J'ai bien l'impression que tu ne nous as pas donné toutes les informations : tu dis arriver à minimiser dans le cas n=2, mais on ne peut pas minimiser $x_1^2\times 3+x_2^2\times (-5)$.

    Cordialement.
  • gerard0
    très bonne remarque, j'ajoute que tous les a1,..,n sont positifs en effet j'ai oublié de préciser
    (même si l'on peut quand même minimiser vôtre exemple il me semble en remplaçant $x_2$ par $1-x_1$ on a un minimum à l'une des bornes de [0 1]).

    [Inutile de reproduire le message précédent. AD]
  • Bonjour,

    Tu n’as toujours pas défini le problème : quelles sont les données, quelles sont les inconnues ?
  • C'est un problème de minimisation sous contraintes : il faut minimiser $\displaystyle (x_1,...,x_n)\mapsto \sum_1^n x_k^2a_k$ sous la contrainte $\sum x_k = 1$.

    Tu peux par exemple exprimer $x_n$ en fonction de $x_1,...,x_{n-1}$ : $x_n=1-\sum_1^{n-1} x_k$ puis remplacer dans $\displaystyle \sum_1^n x_k^2a_k$. Et là tu annules le gradient et tu obtiens un système à résoudre.
  • raoul.S
    oui je pensais en effet que c'est comme cela qu'il faut faire mais le problème c'est que je n'arrive pas à résoudre gradient = 0 dans le cas général ça devient très vite très complexe lorsque j'essaye de développer.
    Merci de vôtre aide !

    [Inutile de reproduire le message précédent. AD]
  • Bonjour,

    Soit des réels positifs $a_k$ pour $k=1,...,n$ pour $n\geq 1$ un entier. On cherche des réels positifs $x_k$ tels que $\sum_k x_k=1$ qui minimisent la somme $\sum_k x_k^2 a_k.$

    Pour tout $a_k=0$, on choisit $x_k=0$ : ce choix ne contribue pas à la somme $\sum_k x_k=1$.

    On peut alors considérer $a_k\neq 0$ sans perte de généralité.

    Par Hoelder ou Cauchy on a $\sum x_k^2 a_k\geq {(\sum_k x_k)^2\over \sum_k 1/a_k}={1\over \sum_k 1/a_k}.$

    Il suffit de montrer que le minimum est atteint.

    Je te laisse terminer en considérant $x_k={y_k \over {\sum_k 1/a_k}}$ et en déterminant les $y_k$ qui réalisent l’égalité. Vérifie la contrainte $\sum_k x_k=1.$
  • C'est très pénible de devoir deviner un énoncé, Louis293. Non seulement tu n'avais pas signalé la positivité des $a_i$, mais voilà maintenant que tu nous sors un "on a un minimum à l'une des bornes de [0 ,1]", intervalle dont tu n'avais jamais parlé. Donc encore une hypothèse manquante (par exemple la positivité aussi des $x_i$).
    Tu fais tout pour qu'on perde notre temps !
  • YvesM
    Merci beaucoup !
    Je pense avoir finalement réussi avec la méthode du gradient qui s'annule comme dit plus haut mais vôtre méthode me permet de vérifier mon résultat car avec ma méthode et la vôtre je trouve la même chose je pense, merci !

    [Inutile de reproduire le message précédent. AD]
  • Bonjour, juste un petit feedback de ma solution si ça peut intéresser certains, pour $i$ compris entre 1 et $N$ on a :
    $$x_i = \dfrac{\prod_{j=1;j \neq i}^{j=N}{a_j}}{\sum_{j=1}^{j=N}{(\prod_{k=1; k \neq j}^{k=N}{a_k})}}.
    $$ Voici un exemple pour rendre la notation plus clair, si $N=3$ alors par exemple $\quad
    x_2 = \dfrac{a_1 a_3}{a_2 a_3 + a_1 a_3+a_1 a_2}$
    On peut vérifié que la somme des x est bien égale à 1

    Je m'excuse si le problème n'était pas clair dès le début, en fait c'est issu d'un problème de base que j'avais commencé à résoudre jusqu'à arriver au sous-problème que j'ai posé ici donc je n'ai pas recopié l'énoncé de base mais essayé de seulement poser moi-même le problème là où j'étais arrivé, bonne journée et encore merci B-)
  • Bonjour,

    Je ne trouve pas ton résultat.

    Je trouve $\displaystyle x_j=0$ pour $\displaystyle a_j=0$ et $\displaystyle x_k = {1/a_k \over \sum_p 1/a_p}.$

    Soit par l'inégalité de Cauchy et en montrant que ces $\displaystyle x_k$ vérifient l'égalité.

    Soit par Lagrange : $\displaystyle F(x) = \sum_p x_p^2 a_p - \lambda (\sum_p x_p-1).$ On calcule $\displaystyle \partial_{x_k} F(x) = 2 x_k a_k - \lambda .$ On somme ces relations $\displaystyle \sum_p x_p = 1 = {\lambda \over 2} \sum_p 1/a_p.$ On en tire $\lambda$ puis $x_k.$

    A moins que tu ne changes l'énoncé ?
  • YvesM écrivait:
    > Je ne trouve pas ton résultat.
    > Je trouve $\displaystyle x_j=0$ pour $\displaystyle a_j=0$ et $\displaystyle x_k = {1/a_k \over \sum_p 1/a_p}.$

    Il me semble que c'est le même résultat que le mien en fait, essayer avec $N=3$, développer $x_2$ et vous avez la même réponse que moi si je ne me trompe pas.
    $x_2 = \dfrac{1/a_2}{\sum_{i=1}^{i=3}{1/a_i}}=\dfrac{a_1 a_3}{a_2 a_3 + a_1 a_3+a_1 a_2}.$
Connectez-vous ou Inscrivez-vous pour répondre.